Five students—the females Ann and Betty and the males Clint, Douglas, and Edward—are rated equally qualified for admission to law school, ahead of other applicants. However, all but two positions have been filled for the entering class. The admissions committee can admit only two more students, so it decides to randomly select two of these five candidates. For this strategy, let of females admitted. Using the first letter of the name to denote a student, the different combinations that could be admitted are (A, B), (A, C), (A, D), (A, E), (B, C), (B, D), (B, E), (C, D), (C, E), and (D, E).
(a)Construct the probability distribution for .
(b)Construct the sampling distribution of the sample proportion of the students selected who are female.

Answers

Answer 1

Answer:

(a) The probability distribution for the number of female students admitted can be calculated as follows:

P(0 female students) = (2C0) × (3C2) / (5C2) = (1) × (3) / (10) = 0.3

P(1 female student) = (2C1) × (3C1) / (5C2) = (2) × (3) / (10) = 0.6

P(2 female students) = (2C2) × (3C0) / (5C2) = (1) × (1) / (10) = 0.1

(b) The sampling distribution of the sample proportion of female students selected can be calculated as follows:

P(p = 0) = 0.3 (for 0 female students)

P(p = 1/2) = 0.6 (for 1 female student)

P(p = 1) = 0.1 (for 2 female students)

Note that the sampling distribution of the sample proportion is a discrete probability distribution with values of 0, 1/2, and 1, and corresponding probabilities 0.3, 0.6, and 0.1, respectively.


Related Questions

A student receives the following​ grades, with an A worth 4 points, a B worth 3​ points, a C worth 2​ points, and a D worth 1 point. What is the​ student's weighted mean grade point​ score?
A. B in 3 three​-credit classes
B. D in 1 three​-credit class
C. in 1 four​-credit class
D. C in 1 two​-credit class

Answers

Weighed averages show that the student has a grade point average of 2.33.

How is the grade point determined?

The grade point average (GPA) is calculated by multiplying the unit value for each course in which a student receives one of the aforementioned grades by the grade point total for that grade. Divide the total of these products by the total number of units. By dividing the total grade points by the total number of units, the cumulative GPA is determined.

A. The student's weighted mean grade point score is (3 classes) × (3 credits per class) × (3 points for a B) ÷ (9 total credits) = 3.00.

B. The student's weighted mean grade point score is (1 class) × (3 credits) × (1 point for a D) ÷ (3 total credits) = 1.00.

C. The student's weighted mean grade point score is (1 class) × (4 credits) × (2 points for a C) ÷ (4 total credits) = 2.00.

D. The student's weighted mean grade point score is (1 class) × (2 credits) × (2 points for a C) ÷ (2 total credits) = 2.00.

To know more about averages visit:-

https://brainly.com/question/14750970

#SPJ1

answer all pls get 100 pointa

Answers

a).The function1 changes at greater rate of 45.

b). At x = 1; the function1 = 45 and function2 = 50.

c).The functions are equal to 90 at x = 2.

What is a function in the form y = mx + c

A function in the equation form "y = mx + c" represents a straight line in a two-dimensional coordinate system. "y" and "x" are variables representing the coordinates of points on the line, "m" is the slope of the line, and "c" is the y-intercept, which is the point at which the line crosses the y-axis.

The slope "m" represents the rate at which the line rises or falls as x increases. The y-intercept "c" determines the location of the line along the y-axis. This equation is known as the slope-intercept form of a linear equation.

For the function1, when y = 0 and x = 0, we can solve for c and m as follows:

0 = 0m + c

c = 0

when y = 90 and x = 2;

90 = 2m + 0

m = 45

Hence, the function1 with y = 45x changes at a greater rate of 45.

At x = 1:

for function1;

y = 45(1) + 0 = 45

for function2;

y = 40(1) + 10 = 50.

At the point when x = 2:

for function1;

y = 45(2) + 0 = 90

for function2;

y = 40(2) + 10 = 90.

Therefore, the function1 changes at greater rate of 45. At x = 1; the function1 = 45 and function2 = 50. The functions are equal to 90at x = 2.

Know more about function here:https://brainly.com/question/24148225

#SPJ1

The scores earned in a flower-growing competition are represented in the stem-and-leaf plot.



0 5
1 0, 3, 7
2 4, 6, 8
3 2
4
5 8
Key: 5|8 means 58


What is the appropriate measure of variability for the data shown, and what is its value?
The range is the best measure of variability, and it equals 18.5.
The IQR is the best measure of variability, and it equals 45.
The range is the best measure of variability, and it equals 45.
The IQR is the best measure of variability, and it equals 18.5.

Answers

The IQR is the best measure of variability, and it equals 45.

What is Variability of data?

Data variation can be caused by a variety of sources. Variability is also known as data spread. The standard deviation is a measure of variability since it measures the spread of data.

The range is the best measure of variability for the data shown, and its value is 45.

The range is defined as the difference between the largest and smallest values in a set of data.

In this case, the largest score is 88 and the smallest score is 20, so the range is 88 - 20 = 68.

The IQR (interquartile range) is another measure of variability, but it is not the best measure for this data set because the data set has only a few values and does not form a normal distribution. The IQR is a more appropriate measure for larger data sets that form a normal distribution.

To Know more about Variability of data visit,

https://brainly.com/question/12872866

#SPJ1

four times the sum of the three consecutive even intergers is 240 more than six times the smallest integer.​

Answers

Answer:

Step-by-step explanation:

Let's call the smallest of the three consecutive even integers "x". Since they are consecutive and even, we know that the next two integers must be "x + 2" and "x + 4".

Now we can set up the equation for the problem:

4 * (x + (x + 2) + (x + 4)) = 240 + 6x

Expanding the parentheses on the left side:

4 * (3x + 6) = 240 + 6x

Simplifying the left side:

12x + 24 = 240 + 6x

Subtracting 6x from both sides:

6x + 24 = 240

Subtracting 24 from both sides:

6x = 216

Dividing both sides by 6:

x = 36

So the smallest of the three consecutive even integers is 36, and the others are 38 and 40.

Answer is x = 36

Three even integers are
x = 36
x + 2 = 38
x + 4 = 40

Step by step

We know even integers = x, x + 2, x + 4

Our equation is

4(x + x + 2 + x + 4) = 6x + 240
Combine like terms in parentheses

4 (3x + 6) = 6x + 240
Multiply

12x + 24 = 6x + 240
Subtract 6x from both sides to combine variables

12x - 6x + 24 = 6x - 6x + 240
Simplify

6x + 24 = 240
Subtract 24 from both sides to combine constants

6x + 24 - 24 = 240 - 24
Simplify

6x = 216
Divide both sides by 6 to solve

6/6 x = 216/6

x = 36
x + 2 = 38
x + 4 = 40

Check your work using the original equation

4(x + x + 2 + x + 4) = 6x + 240

4 (36 + 36 + 2 + 36 + 4) = 6(36) + 240
Combine like terms inside parentheses

4(114) = 6(36) + 240
Multiply

456 = 456

They equal so solution is correct



Grayson is going to invest in an account paying an interest rate of 4% compounded
continuously. How much would Grayson need to invest, to the nearest dollar, for the
value of the account to reach $15,000 in 5 years?

Answers

Grayson would need to invest approximately $10,994 to the nearest dollar for the value of the account to reach $15,000 in 5 years.

What is continuously compounded interest?

Continuous compounded interest indicates that an account balance is constantly earning interest and reinvesting that money so that it, too, earns interest.

The formula for continuous compounding is given by:

[tex]A = Pe^{rt}[/tex]

where A is the amount in the account after t years, P is the principal amount (the amount invested), r is the annual interest rate, and e is the base of the natural logarithm.

We want to find P when A = $15,000, r = 0.04, and t = 5 years.

Substituting these values into the formula, we get:

[tex]15000 =Pe^{0.04*5}[/tex]

Simplifying the right-hand side, we get:

[tex]15000 =Pe^{0.2}[/tex]

Dividing both sides by e^(0.2), we get:

[tex]P= \frac{15000}{e^{0.2}}[/tex]

Using a calculator, we get:

P ≈ $10,994

Therefore, Grayson would need to invest approximately $10,994 to the nearest dollar for the value of the account to reach $15,000 in 5 years.

To know more about continuously compounded interest visit,

https://brainly.com/question/14303868

#SPJ1

Qué fracción restada a 1/2 da como resultado 4/10?

Answers

-1/10 should be subtracted from 1/2 to get 4/10.

How can you represent a fraction? Are fractions and ratios same?

A fraction is of the form → (x/y).

For example, we can write fractions as → 2/3.

Fractions and ratios represent the same aspect. We can write -

2 : 3 = 2/3

4 : 7 = 4/7

Given is to solve the expression -

(1/2) - ? = 4/10

We can write the expression as -

(1/2) - (x) = (4/10)

(x) = 4/10 - 1/2

(x) = 4/10 - 5/10

(x) = -1/10

Therefore, -1/10 should be subtracted from 1/2 to get 4/10.

To solve more questions on subtracting fractions, visit the link below

https://brainly.com/question/30489902

#SPJ9

{Question in English -

What fraction subtracted from 1/2 equals 4/10?}

8m
6m
10m
5m what is the area of this shape

Answers

Answer:

68 m²

Step-by-step explanation:

we have two rectangles side by side, the larger one measures 8m by 6m, the smaller one 5m by 4m, the 4 meters is the difference between the base  (10m) and the 6m side, so we find the areas and add them using the formula A = L x W, the result is not in the choices but it is right.

6 * 8 + 4 * 5 =                     remember PEMDAS

48 + 20 =

68 m²

I need help answering this

Answers

The domain of the function for this situation is given as follows:

B. All whole numbers greater than or equal to 0 and less than or equal to 7.

How to obtain the domain of a function?

The domain of a function is the set that contains all the values assumed by the input of the function.

Hence, on a graph, the domain is given by the values assumes on the horizontal axis of the graph.

The values that compose the domain of the graph are given as follows:

0, 1, 2, 3, 4, 5, 6 and 7.

Which is best described by option B.

More can be learned about domain of functions at https://brainly.com/question/30348593

#SPJ1

BRAINLIEST if correct, 5th grade geometry

Answers

Answer:

EF, GH

Step-by-step explanation:

both go on forever from one point

Answer:

A and B

Step-by-step explanation:

Because they go infinitly in one direction and end in another.

The figure below is dilated by a factor of 4 centered at the origin. Plot the resulting image

Answers

The figure below has been dilated by a factor of 4 centered at the origin. The resulting image can be plotted using the coordinates (x', y'):

(x', y') = (4x, 4y)

For example, if the original coordinate point is (3, 5), then the new coordinate point would be (12, 20).

The resulting image of the dilated figure has vertices at (-8, -4), (8, -8), and (4, 8).

What is dilation?

Resizing an item uses a transformation called dilation. Dilation is used to enlarge or shorten the structures. The result of this transformation is an image with the same shape as the original. However, there is a variation in the shape's size. The initial form should be stretched or contracted during dilatation.

To dilate the given figure by a factor of 4 centered at the origin, we need to multiply the coordinates of each vertex by 4.

The new coordinates of B will be:

x-coordinate: 4 x (-2) = -8

y-coordinate: 4 x (-1) = -4

So the new vertex B' is (-8, -4).

The new coordinates of C will be:

x-coordinate: 4 x (2) = 8

y-coordinate: 4 x (-2) = -8

So the new vertex C' is (8, -8).

The new coordinates of D will be:

x-coordinate: 4(1) = 4

y-coordinate: 4(2) = 8

So the new vertex D' is (4, 8).

Therefore, the image of the resultant figure is given in the image below.

To learn more about the dilation in geometry;

https://brainly.com/question/10713409

#SPJ2

NAME
12. Ling wants to buy a sleeping bag.
The sale price is $9. What was the
original price if the sale price was 30?

Answers

discount = original price - sale price
original price = sale price + discount
original price = $9 + ($30 - $9)
original price = $9 + $21
original price = $30

Therefore, the original price of the sleeping bag was $30, before the discount was applied and the price was lowered to $9.

A magician shuffles a standard deck of playing cards and allows an audience member to pull out a card, look at it, and replace it in the deck. Three additional people do the same. Find the probability that of the 4 cards drawn, at least 1 is a face card. (Round your answer to one decimal place.

Answers

The probability that at least 1 of the 4 cards drawn is a face card is 0.6.

How to find probability of 4 cards drawn?

A typical deck contains 52 cards in total. There are 51 cards left in the deck after the first spectator draws a card. The replacement card leaves 52 cards in total, which is the same as before. The number of cards remains 52 for all four draws since each succeeding audience member will draw a card and replace it.

Finding the likelihood that none of the four cards are face cards and subtracting it from one will give us the chance that at least one of the four cards is a face card.

A deck of cards has 12 face cards (4 jacks, 4 queens, and 4 kings). Consequently, the likelihood that a single draw will not provide a face card is:

P(not a face card)=(52 - 12)/(52)=(40/(52)=(10/13).

The likelihood that none of the four draws will produce a face card is:

P(no face cards are present) = (10/13)4 = 0.432

As a result, the likelihood that at least 1 of the four cards revealed is a face card is as follows:

P(none are face cards) = 1 - P(at least one is a face card) = 1 - 0.432 = 0.568

The chance, rounded to one decimal place, is roughly 0.6.

Consequently, there is a 0.6 percent chance that at least one of the four cards will be a face card.

To know more about Standard Deck visit:

brainly.com/question/13935092

#SPJ1

What is 7/8 of sixteen times a half of 30

Answers

7/8 of sixteen times a half of 30 is 210.

What is a PEMDAS rule?

PEMDAS is an acronym that stands for the order of operations in mathematics: Parentheses, Exponents, Multiplication and Division (from left to right), and Addition and Subtraction (from left to right).

To calculate this, we can follow the order of operations, which is also known as PEMDAS (Parentheses, Exponents, Multiplication and Division, Addition and Subtraction):

Start with the expression inside the parentheses: half of 30 is 15.

Multiply 16 by 15: 16 times 15 is 240.

Find 7/8 of 240:

7/8 of 240 = (7/8) x 240

                  = 210

Hence, 7/8 of sixteen times a half of 30 is 210.

To learn more about the PEMDAS rule, visit:

https://brainly.com/question/23385237

#SPJ1

what happend if you do rise over run

Answers

PLEASE GIVE BRAINLIEST
thank you and have a good day :)

Answer:

You figure out the slope

Step-by-step explanation:

Rise/Run is a formula that calculates the slope.  This determines the direction of the line and how steep it is.

Answer:

its a math term yw my boy

Step-by-step explanation:

Which of the following shows a correct method to calculate the surface area of the cylinder?

cylinder with diameter labeled 2.6 feet and height labeled 4.4 feet

SA = 2π(2.6)2 + 2.6π(4.4) square feet
SA = 2π(2.6)2 + 1.3π(4.4) square feet
SA = 2π(1.3)2 + 1.3π(4.4) square feet
SA = 2π(1.3)2 + 2.6π(4.4) square feet

Answers

Option D is correct, SA = 2π(1.3)2 + 2.6π(4.4) square feet is used calculate the surface area of the cylinder.

What is Three dimensional shape?

a three dimensional shape can be defined as a solid figure or an object or shape that has three dimensions—length, width, and height.

Given that cylinder with diameter labeled 2.6 feet and height labeled 4.4 feet

The surface area of cylinder is given by the formula =2πrh+2πr²

The radius is Diameter/2

2.6/2=1.3

The radius is 1.3

Surface area =2π(1.3)(4.4)+2π(1.3)²

So =2.6π(4.4)+2π(1.3)²

Hence, Option D is correct, SA = 2π(1.3)² + 2.6π(4.4) square feet is used calculate the surface area of the cylinder.

To learn more on Three dimensional figure click:

https://brainly.com/question/2400003

#SPJ1

Answer:

Step-by-step explanation:

Find the sum of the infinite geometric series, if possible. (Round your answer to three decimal places.)

Σ
n = 0 6(0.75)n
STEP 1: Determine the first term of the infinite geometric series.
a1 =
24

STEP 2: Find the common ratio of the infinite geometric series.
r =

STEP 3: Based on your results from Step 1 and Step 2, the Sum of an Infinite Geometric Series Formula may be applied to this series. What guarantees the formula will work in this case?
The common ratio has an absolute value less than 1.
The first term of the series has an absolute value less than 1.
The common ratio has an absolute value less than the first term of the series.
The common ratio is less than the first term of the series.
The common ratio is less than one.
The first term of the series has an absolute value greater than the common ratio.
The first term of the series is positive.

STEP 4: Compute the sum of the series using the Sum of an Infinite Geometric Series Formula. Round your answer to three decimal places.
Sn =

Answers

STEP 1: a1 = 6.

STEP 2:  r = 0.75

STEP 3: The common ratio is less than one.

STEP 4: The sum of the  Sum of the Infinite Geometric series is Sn = 24

How to find the sum of the infinite geometric series?

Since we have the expression for the sum of the infinite geometric series

[tex]\[ \sum_{n=0}^{\infty} 6(0.75)^{n} \][/tex]

STEP 1:

The first term of the infinite geometric series is 6. Thus, a1 = 6.

STEP 2:

The common ratio of the infinite geometric series is 0.75. Thus, r = 0.75

STEP 3:

The common ratio is less than one.

Because Sn = a/(1 - r)  is the formula we use when the common ratio is less than 1.

STEP 4:

Sn = a/(1 - r)

Sn = 6/(1 - 0.75) = 24

Learn more about sum of infinite geometric series on:

https://brainly.com/question/26255015

#SPJ1

someone pls help with this question
a² y² - 2aby + b²​

Answers

The expression you provided is a quadratic equation in the form of (a^2)y^2 - 2aby + b^2.

This equation can be factored into the form of (ay - b)^2, which is a perfect square trinomial. This means that it can be written as the square of a binomial, (ay - b).

So, the factored form of the equation is:

(ay - b)^2 = (a^2)y^2 - 2aby + b^2

This form shows that the expression is the square of a binomial and can be used to solve for y.

I will give brainliest and ratings if you get this correct ​

Answers

The short-run total cost is TC is wL + rK and short-run average cost curve is TC/Q. The SMC is 4.01 and the SMC curve intersects the SAC curve at its lowest or minimum points because the average cost is a weighted average of the marginal cost and the fixed cost.

What is the firm short-run total cost curve and short-run average cost curve function

A. To calculate the firm’s short-run total cost curve, we need to calculate the total cost at each level of output. The short-run total cost can be represented as follows:

TC = wL + rK,

where w is the wage rate, L is the labor used, r is the interest rate, and K is the capital used. In this case, w = $4 and r = $1.

To calculate the short-run average cost curve, we need to divide the total cost by the level of output:

SAC = TC / Q

Where Q is the level of output.

B. The short-run marginal cost function can be calculated as the change in total cost for a small change in the level of output:

SMC = dTC / dQ

At a level of output of 25, the labor used is L = 25 / 2 = 12.5. The total cost can be calculated as follows:

TC = 4 * 12.5 + 1 * 100 = 50 + 100 = 150

So, the short-run average cost is SAC = 150 / 25 = $6 and the short-run marginal cost is SMC = dTC / dQ. To calculate the derivative, we need to use the production function given:

Q = 2 * sqrt(KL)

Taking the derivative with respect to L, we get:

dQ / dL = (1 / (2 * sqrt(KL))) * 2K = K / (sqrt(KL))

Substituting the values, we get:

SMC = 4 + (1 / (sqrt(100 * 12.5)))

At a level of output of 200, the labor used is L = 200 / 2 = 100. The total cost can be calculated as follows:

TC = 4 * 100 + 1 * 100 = 400

So, the short-run average cost is SAC = 400 / 200 = $2 and the short-run marginal cost is SMC = dTC / dQ. To calculate the derivative, we need to use the production function given:

Q = 2 * sqrt(KL)

Taking the derivative with respect to L, we get:

dQ / dL = (1 / (2 * sqrt(KL))) * 2K = K / (sqrt(KL))

Substituting the values, we get:

SMC = 4 + (1 / (sqrt(100 * 100)))

SMC = 4.01

C. To draw the graph for the SAC and SMC, we need to calculate the SAC and SMC for different levels of output. We can then plot the SAC and SMC against the level of output to obtain the graphs.

D. The SMC curve intersects the SAC curve at its lowest or minimum points because the average cost is a weighted average of the marginal cost and the fixed cost. When the level of output is low, the marginal cost is high and the fixed cost is a large portion of the total cost. As the level of output increases, the marginal cost decreases and the fixed cost becomes a smaller portion of the total cost. As a result, the average cost decreases and reaches a minimum at the point where the marginal cost intersects the average cost.

Learn more on short-run total cost curve here;

https://brainly.com/question/13694840

https://brainly.com/question/25109150

https://brainly.com/question/13743514

#SPJ1

Which is true about the function y=−2x?

It is a nonlinear function because its graph passes through the origin.

It is a nonlinear function because its graph is a straight line.

It is a linear function because its graph passes through the origin.

It is a linear function because its graph is a straight line.

Answers

The answer choice which is true about the given equation; y = -2x as required to be determined is; It is a linear function because its graph is a straight line.

Which answer choice represents a true statement?

It follows from the task content that the given function y = -2x as required be identified as being a linear equation or not and why?

On this note, since the graph of the equation is such that g

it yields a straight line, it follows that the given function; is a linear function because its graph is a straight line.

Read more on linear functions;

https://brainly.com/question/4025726

#SPJ1

Find the area of the figure pictured below.

Answers

The area of the composite figure is 41 ft square.

How to find the area of a figure?

The figure is a composite figure. A composite figure is a figure that has two or more shapes.

Therefore, the area of the composite figure is the sum of the individual area of the shapes.

Therefore,

area of the figure = area of rectangle 1 + area of rectangle 2

area of rectangle 1 = lw

where

l = lengthw = width

Therefore,

area of rectangle 1 = 2 × 4 = 8 ft²

area of rectangle 2 = 11 × 3 = 33 ft²

Hence,

area of the figure =  = 8 + 33

area of the figure = 41 ft²

learn more on area here: https://brainly.com/question/23718948

#SPJ1

Rewrite the equation in Ax+By=C form
y-6=5(x+2)

Answers

Put, y-6=5(x+2) into the form,Ax+By=C.


y-6=5(x+2), distribute the 5 into the parenthesis

=> y-6=5x+10, add the value of 6 to both sides

=> y=5x+16, subtract the value 5x from both sides

=> y-5x=16, thus we are in the form Ax+By=C

Question 10 Part 1 of 3

When a bactericide is added to a nutrient broth in which bacteria are growing, the bacterium
population continues to grow for a while, but then stops growing and begins to decline. The size of
the population at time t (hours) is b=8^6 +8^3t-8^2 t^2. Find the growth rates at t=0 hours,
t=4 hours, and t=8 hours.
The growth rate at t=0 hours is _____bacteria per hour

Answers

The population at 0 hours is 1,000, at 4 hours is 1,000,024, and at 8 hours is  1,000,016.

How to calculate the number of bacteria over time?

To calculate how the number of bacteria changes over time, let's use the function provided and replace the variable t = time.

0 hours:

Population = 10^6 + (10^4)0 -(10^3)0^2

Population = 10^6 - 10^3

Population = 10^3 or 1,000

4 hours:

Population = 10^6 + (10^4)4 -(10^3)4^2

Population= 10^6 + 40,000 - 16,000

Population= 1,000,024

8 hours:

Population = 10^6 + (10^4)8 -(10^3)8^2

Population= 10^6 + 80,000 - 64,000

Population= 1,000,016

Learn more about growth rate in https://brainly.com/question/13870574

#SPJ1

Enter each answer as a whole number or a faction, or DNE forDoes Not Exist

Answers

The values of the piecewise functions at the indicated limits using the equations obtained from the graph are;

[tex]\lim \limits_{x \to2^+}\frac{f(x) - 4}{f(x+ 3)} = \underline{-\frac{2}{3} }[/tex]

[tex]\lim \limits_{x\to 3^-}[/tex] f(f(x) + 4) = 7

[tex]\lim\limits_{h\to 0} \frac{f(2+ h)-f(2)}{h}[/tex] DNE

What is a piecewise function?

A piecewise function is a function that has rules or definition based on the interval of reference of the function.

The graphed function, f(x), comprising of several piecewise functions, indicates that as x → 2⁺, we get;

The points on the line of the graph are;

(1, 1), and (3, 3)

The slope of the graph as x tends to 2⁺ is therefore;

Slope = (3 - 1)/(3 - 1) = 1

The equation of the line is therefore;

y - 1 = x - 1

y = f(x) =  x

Therefore;

f(x) - 4 = x - 4

f(x + 3) = x + 3

[tex]\lim \limits_{x\to 2^+} \frac{f(x) -4}{f(x + 3)} = \frac{x - 4}{x + 3} | x = 2[/tex]

[tex]\lim \limits_{x\to 2^+} \frac{f(x) -4}{f(x + 3)} = \frac{2 - 4}{2 + 3} =\underline{-\frac{2}{5}}[/tex]

The piecewise function for the interval, x → 3⁻ is; f(x) = x, therefore;

f(f(x) + 4) = f(x + 4) = x + 4

[tex]\lim \limits_{x\to 3^-}f(f(x) + 4)[/tex] = x + 4, where; x = 3, therefore;

[tex]\lim \limits_{x\to 3^-}f(f(x) + 4)[/tex] = 3 + 4 = 7

[tex]\lim \limits_{h\to 0} = \frac{f(2 + h)-f(2)}{h}[/tex], can be found as follows;

The equation of the function at x = 2 is; y = f(x) = x

f(2 + h) = 2 + h

f(2) = 2

Therefore;

[tex]\frac{f(2 + h)-f(2)}{h}[/tex] = (2 + h - 2)/h = h/h

[tex]\lim \limits_{h\to 0} \frac{f(2 + h)-f(2)}{h} = \frac{0}{0}[/tex] DNE (Does Not Exist)

Learn more about the piecewise functions here: https://brainly.com/question/14390119

#SPJ1

What is the measure of each angle in a regular polygon with 14 sides? If necessary, round your answer to the nearest tenth.

Answers

The measure of each angle in the regular polygon is 154.3 degrees

How to determine the measure of each angle in a regular polygon

From the question, we have the following parameters that can be used in our computation:

Number of sides, n = 14

The measure of each angle is calculated as

Angle = ( n − 2 ) × 180/n

substitute the known values in the above equation, so, we have the following representation

Angle = (14 − 2) × 180/14

Evaluate

Angle = 154.3

Hence, the angle is 154.3

Read more about angles at

https://brainly.com/question/25716982


#SPJ1

the equation of a curve is given by
y=x^3/3+7x^2/2+10x+d, where d is a
constant. Find the possible values of d
when the x-axis is tangent to the curve
v=x^3/3+7x^2/2+10x+d.

Answers

Answer: A tangent to the x-axis means that the y-coordinate of the point of tangency is 0. So, we can find the possible values of d by setting y = 0 and solving for x:

0 = x^3/3 + 7x^2/2 + 10x + d

This is a cubic equation, so it can be solved using various methods such as factoring, completing the square, or using a numerical method such as the Newton-Raphson method. If the equation can be factored or the method used is numerical, there might be multiple possible values of d. If the method used is completing the square, there would be only one possible value of d.

However, without a specific method, it's not possible to find the exact value(s) of d. The possible values of d will depend on the specific solution method used to solve the equation.

Step-by-step explanation:

Scenario #1: Raul.

raul is a saver. he sets aside $100 per month during his career of 40 years to prepare for retirement. He does not like the idea of investing because he prefers to minimize his risk as much as possible so he puts his money in a savings account, which earns 1.5% interest per year.

1. what is the total balance in the account after 40 years?

2. how much of a total did Raul contribute himself?

3. how much money did Raul make through compound interest in this savings account?

4. identify one way Raul could have increased the total amount of money he made over 40 years. Explain your reasoning..

Answers

After 40 years, the account has a total balance of $49.450.80.

Why is there an interest?

Any loans or borrowings come with interest. the portion of the loan's value that lenders use to calculate interest. By lending money (via a bond or certificate of deposit, for instance), or by depositing funds into an interest-bearing bank account, consumers can earn interest.

Let's assume that he starts with $0 in the account and makes a monthly contribution of $100 for 40 years, which is equal to 40 x 12 = 480 months.

The total contributions would be $100 x 480 = $48,000.

The interest earned on the account balance can be calculated as follows:

Interest = Account balance * Interest rate

At the end of each year, the interest is added to the account balance. So, after 40 years, the balance would be:

Year 1: $48,000 * 1.5% = $720

Balance = $48,000 + $720 = $48,720

Year 2: $48,720 * 1.5% = $730.80

Balance = $48,720 + $730.80 = $49,450.80

To know more about interest visit:-

https://brainly.com/question/11339060

#SPJ1

what is the equation of a line that is parallel to the line that is given by the equation x=-4?

Answers

The equation of the line which is parallel to the line x = -4 will be x = -3.

What is an equation of the line?

An equation of the line is defined as a linear equation having a degree of one. The equation of the line contains two variables x and y. And the third parameter is the slope of the line which represents the elevation of the line.

The general form of the equation of the line:-

y = mx + c

m = slope

c = y-intercept

Slope = ( y₂ - y₁ ) / ( x₂ - x₁ )

Given that the equation of the line is x = -4. The y-intercept is zero for the line.

The equation of the line parallel to the equation will be,

x = -3

To know more about an equation of the line follow

https://brainly.com/question/18831322

#SPJ9

The weight of a small Starbucks coffee is a normally distributed random variable with a mean of 400 grams and a standard deviation of 20grams. Find the weight that corresponds to each event.
(Use Excel or Appendix C to calculate the z-value. Round your final answers to 2 decimal places.)
a. Highest 20 percent
b. Middle 60 percent to
c. Highest 80 percent
d. Lowest 15 percent.​

Answers

Answer:

a. To find the weight corresponding to the highest 20 percent of coffees, we need to find the z-score that corresponds to the 0.8 cumulative probability.

Using a standard normal table or a calculator that can compute z-scores, we can find that the z-score corresponding to a cumulative probability of 0.8 is 0.8416.

Next, we use the z-score formula to find the weight:

Weight = Mean + (z-score * Standard Deviation)

Weight = 400 + (0.8416 * 20) = 448.32 g

So, the weight corresponding to the highest 20 percent of coffees is approximately 448.32 g.

b. To find the weight corresponding to the middle 60 percent of coffees, we need to find the z-scores that correspond to the cumulative probabilities of 0.2 and 0.8.

Using a standard normal table or a calculator that can compute z-scores, we can find that the z-score corresponding to a cumulative probability of 0.2 is -0.8416, and the z-score corresponding to a cumulative probability of 0.8 is 0.8416.

Next, we use the z-score formula to find the weights:

Weight Lower Bound = Mean + (z-score * Standard Deviation)

Weight Lower Bound = 400 + (-0.8416 * 20) = 351.68 g

Weight Upper Bound = Mean + (z-score * Standard Deviation)

Weight Upper Bound = 400 + (0.8416 * 20) = 448.32 g

So, the weight corresponding to the middle 60 percent of coffees ranges from approximately 351.68 g to approximately 448.32 g.

c. To find the weight corresponding to the highest 80 percent of coffees, we use the same process as in (a) to find the z-score that corresponds to the cumulative probability of 0.8, which is 0.8416.

Next, we use the z-score formula to find the weight:

Weight = Mean + (z-score * Standard Deviation)

Weight = 400 + (0.8416 * 20) = 448.32 g

So, the weight corresponding to the highest 80 percent of coffees is approximately 448.32 g.

d. To find the weight corresponding to the lowest 15 percent of coffees, we need to find the z-score that corresponds to the cumulative probability of 0.15.

Using a standard normal table or a calculator that can compute z-scores, we can find that the z-score corresponding to a cumulative probability of 0.15 is -0.9332.

Next, we use the z-score formula to find the weight:

Weight = Mean + (z-score * Standard Deviation)

Weight = 400 + (-0.9332 * 20) = 346.64 g

So, the weight corresponding to the lowest 15 percent of coffees is approximately 346.64 g.

Step-by-step explanation:

A bag of garden soil weighs 42 pounds and holds 2 cubic feet. Find the weight of 15 bags in kilograms and the volume of 15 bags in cubic yards.

Answers

Answer:

285.76 kilograms       1.1 cubic yard

Step-by-step explanation:

Answer:

1.11121 cubic yards.

Step-by-step explanation:

First, let's convert the weight of 1 bag of garden soil from pounds to kilograms:

1 lb = 0.45359237 kg

42 lbs = 42 * 0.45359237 kg = 19.05 kg

So, the weight of 1 bag of garden soil is 19.05 kilograms.

Next, let's find the weight of 15 bags of garden soil:

15 bags * 19.05 kg/bag = 285.75 kg

So, 15 bags of garden soil weigh 285.75 kilograms.

Next, let's find the volume of 15 bags of garden soil in cubic yards:

1 cubic foot = 0.037037 cubic yards

2 cubic feet * 15 bags = 30 cubic feet

30 cubic feet = 30 * 0.037037 cubic yards = 1.11121 cubic yards

So, 15 bags of garden soil have a volume of 1.11121 cubic yards.

Select the correct answer.
A fantasy board game makes use of four dice. The first is a standard 6-faced die, the second die has 4 faces, and the other two have 10 faces each. If all the dice are thrown together and their combination dictates a certain outcome of the game, how many possible outcomes exist?

A.
30

B.
240

C.
2,400

D.
3,000

Answers

Answer:

The total possible outcomes that exist are:

                            2400

Step-by-step explanation:

We are given four dices:

first is a standard 6-faced die.

The second die has 4 faces.

Third dice has 10 faces.

Fourth dice has 10 faces.

Now when all the dice are thrown together then the total number of possible outcomes are:

 

( Since there are 4 numbers possible for the first die, 6 for second and 10 each for third and fourth)

                   The answer is:

                          2400

Other Questions
The triangle below is equilateral. Find the length of side x in simplest radical form with a rational denominator. x 6 The text presents which two analytic tools to help with decision making?a. decision tables and decision networksb. decision tables and decision treesc. decision networks and decision treesd. decision networks and decision derivations how was georgia significant in the life of president roosevelt? A___is a numerical measure of the outcome of a probability experiment an element has a dibromide with the empirical formula and a dichloride with the empirical formula . the dibromide is converted to the dichloride according to the equation if the complete conversion of 1.238 g of results in the formation of 0.735 g of , what is the atomic mass of the element ? which compound is the most soluble in water? group of answer choices ch3ch2ch2ch2ch2ch2ch2oh ch3ch2ch2ch2oh ch3ch2ch2oh ch3ch2ch2ch2ch2oh all of these compounds are equally soluble in water. Which of the following is a part of the interpersonal communication model that led to the communication breakdown between Jane and the customer?a. inadequate or poor communicationb. busy business environmentc. avoiding blaming the customer Did Sparta treat it citizens differently then Athens What are the main differences between domain bacteria and archaea? (a) the double-strand breaks occur along the dna backbone. describe the process by which the breaks occur. which three values are smaller than 1/2:A. 1/4B. 0.26C. 3/5D. 67%E. 0.74F. 40% The manager of the commercial mortgage department of a large bank has collected data during the past two years concerning the number of commercial mortgages approved per week. The results from these two years(104weeks) are shown to the right.Compute the standard deviation.c. What is the probability that there will be more than one commercial mortgage approved in a given week?Number ApprovedFrequency01312523331749556171 HELPPPP PLS I NEED HELP which method is used to reduce nitrogen oxide emissions? installation of catalytic converters increased temperature of combustion the addition of oxygen to combustion processes the use of fluidized bed combustion which is not used to prevent the emission of particulate matter? n a population with two alleles b and b, the allele frequency of b is 0.7. if this population is in hardy-weinberg equilibrium. what is the frequency of heterozygotes, homozygous dominant, and homozygous recessives? g Use the substitution method to solve the system of linear equations.y = x - 44x + y = 26 Why are employee benefits important to employees? What mass of HCl is contained in 45.0 mL of an aqueous HCl solution that has a density of 1.19g cm ^ - 3 and contains 37.21% HCl by mass? which of the following sdl activities do occur in the design and development phase? question 1 options: threat modeling privacy information gathering and analysis static analysis open source selection if available A high school theatre club has 40 students, of whom 6 are left handed